• AmadeusD
    2k
    He just vanishes into thin air?Luke

    This would be the only realistic result, but then it would follow that this means he was never able to come back to kill Young Bob.

    So we're still stuck with the 'copy' idea.
  • noAxioms
    1.4k
    You said you're creating a new world
    — noAxioms
    I have never said this.
    Luke
    Not exactly in those words:

    there must be two (or more) parallel timelines in order for time travel to make sense. The timelines branch off into two or more timelines following the first time travel event.Luke
    That says a parallel timeline [world] is needed, created since it doesn't otherwise exist. The 2nd sentence implies the 1990 new timeline branches off the 2024 'travel' event, which means no actual travel, just a universe creation event at 2024.
    You say this, but perhaps worded it poorly and meant actual travel to an actually existing 1990, so it isn't something (a whole universe) that you need to manufacture.

    my ontology of time involves a blend of presentism and eternalism (in short, that without presentism there is no 'progression of events',Luke
    OK, you acknowledge that the concept of a timeline implies the lack of presentism. There is no need for a 'progression of events'. Time travel under eternalism simply involves a worldline that is discontinuous, or doesn't follow a timelike path. So we ditch the presentism altogether, and that gives us a 1990 destination which we select as our target.

    Now we do some retro-causality magic and branch a new timeline off of that point, which doesn't alter the original line at all except for Bob's abrupt disappearance from 2024, a violation of a bunch of conservation laws that we are ignoring. How is this Bob in the new timeline the same Bob as the old timeline? I mean, with the usual parallel timelines (MWI here), the Luke in some other world is not you, but somebody else.

    There's no contradictions with it because killing the copy young-Bob isn't killing old-Bob's actual ancestor.
    — noAxioms

    You can call it a "copy" if you like. There are two parallel timelines, after all: one timeline in which Young Bob grows up to build a time machine and travel to 2024 and another in which Young Bob gets killed by Old Bob.
    Luke
    Sounds like a copy to me. Old Bob is a continuation of the not-murdered original young Bob, not the Bob that gets murdered.

    However, what supposedly happens to Old Bob in your single timeline scenario after he murders Young Bob?
    I don't have a single-timeline scenario. Heck, I don't have a scenario at all. Just trying to figure yours out. I've changed my guess significantly based on what you've said and based on some past comments that I read again. Is it better now?
    What was our point here? If we can do this impossible thing, no contradiction need exist (except for the magic in doing it). I've long since expressed that the branching solution resolves the grandfather paradox.

    This would be the only realistic resultAmadeusD
    Sort of like Marty (or his picture of his older siblings) beginning to fade as he slowly destroys any possibility of his parents hooking up. Hollywood loves this idea despite the paradox it creates.
  • Luke
    2.6k
    there must be two (or more) parallel timelines in order for time travel to make sense. The timelines branch off into two or more timelines following the first time travel event.
    — Luke

    That says a parallel timeline [world] is needed, created since it doesn't otherwise exist.
    noAxioms

    It does exist. It's the same 1990 that was there before (on the original timeline), except that before it did not contain any time travelling Bob. The only difference is that now it does contain a time travelling Bob, because Bob has just time travelled back from 2024 to 1990.

    The 2nd sentence implies the 1990 new timeline branches off the 2024 'travel' event, which means no actual travel, just a universe creation event at 2024.noAxioms

    I wouldn't consider it time travel if a new 1990 must be invented at/in 2024. I consider that Bob's time machine actually works and transports him back to 1990 from 2024. He can't already be in 1990 (at his 2024 age, as Old Bob) without having time travelled.

    The 1990 new timeline does branch off the 2024 travel event, but it creates a new timeline - following the 2024 travel event - which begins at 1990, in which Old Bob has now time travelled back and inserted himself into 1990. He wasn't at 1990 on the original timeline (as Old Bob) because he had not yet built the time machine. Everything else is exactly the same as it was in 1990 upon Old Bob's arrival, except that now it includes Old Bob. Isn't that just what we usually mean by time travel? If I wanted to travel back in time to kill Hitler, presumably I'm not already there/then.

    How is this Bob in the new timeline the same Bob as the old timeline?noAxioms

    Because he was Young Bob on the original timeline and then he grew up (into Old Bob) to build the time machine and used it to time travel back to 1990 on the new timeline to kill Young Bob. It's a logical sequence of events: Young Bob grows into Old Bob who time travels back to 1990 and kills his younger self. Old Bob continues living after this, and Young Bob no longer continues on to build a time machine. That's how the new timeline differs from the old timeline, which is a result of Old Bob's time travel and subsequent actions.

    They are different timelines, but they started out the same way, and it's the same Young Bob in both up until the arrival of Old Bob from the future, which is when the two timelines diverge. However, the original timeline must precede the new, second timeline. Bob can't time travel or return to the past before he has built the time machine.

    Sounds like a copy to me. Old Bob is a continuation of the not-murdered original young Bob, not the Bob that gets murdered.noAxioms

    Right, Old Bob is not the Bob that gets murdered, because Old Bob is the murderer of Young Bob. You could say that Old Bob only murders a copy of Young Bob on the new timeline and that he does not murder the original Young Bob. However, murdering the original Young Bob is not possible according to the logic of the scenario, because the time machine needs to be built before Old Bob can travel to the past in order to kill Young Bob. Bob effectively changes the past from what it was on the original timeline (from 1990 onwards), which was his intent.

    I don't have a single-timeline scenario. Heck, I don't have a scenario at all.noAxioms

    Sorry, I just assumed this from past discussions, since I thought you were firmly on the side of eternalism.

    Is it better now?noAxioms

    Yes, thanks for taking the time.

    I've long since expressed that the branching solution resolves the grandfather paradox.noAxioms

    I assume by "branching solution" you mean something similar to what I've described here. Otherwise, I'm not sure what that means.

    I only just found out yesterday that the idea of parallel worlds is not a new idea as a solution to the grandfather paradox.
  • Corvus
    3k
    Not sure if there was anyone witnessing the Big Bang
    — Corvus
    We did. It's not like it happened a finite distance away and the view of the bang has already passed us by. Of course the really early events are obscured by the opaque conditions back then. The window through which we look took a third of a million years or so to turn transparent. By that measure, nothing could 'see' the big bang since it was all obscured behind a blanket until then.
    noAxioms
    I don't recall seeing it at all. Or anyone who claims to have witnessed the Big Bang.
    Could it happen again? Or is it still happening the now?

    Nothing ever gets proved. I can go to grandma's house if I have a car, and the weather is acceptable, and if I draw breath. But technically I cannot prove any of those.
    Point is, requiring 'proof' is going to far. Evidence of X,Y Z is probably enough for plausible time travel. Right now, that evidence is very negative.
    noAxioms
    There is a difference, when you are just keep talking to yourself making the "If" statements to yourself, and when you actually make philosophical propositions in public claiming that it is true or at least valid and sound.

    Your interlocutors will request to prove your statements, if they sound unclear, invalid and unsound, so they can be rejected. Philosophical debates are about proving and disproving claims and statements as true, false or neither. And a statement cannot be both true and false. That wound be a contradiction.

    The point is that unless it is your monologue or a pub talk, philosophical statements need to be proved and verified when not clear. If not, it could transform to a drama or comedy.


    I am not sure what the physical clock measures.
    — Corvus
    It measures proper time, which is very defined in both interpretations of time. It doesn't measure the advancement of the present, or the rate of the flow of time. That sort of time is more abstract, and there is no empirical way to detect it, let alone measure it. So maybe we're talking past each other when I reference the sort of time that clocks measure, vs you referencing the latter.
    noAxioms
    It sounds like we are talking about different time here. What is "proper time" and "interpretation of time"?
  • AmadeusD
    2k
    Sort of like Marty (or his picture of his older siblings) beginning to fade as he slowly destroys any possibility of his parents hooking up. Hollywood loves this idea despite the paradox it creates.noAxioms

    Agreed, more or less.
  • Luke
    2.6k
    Sounds like a copy to me.noAxioms

    Having given this some further consideration, although some may find it unsatisfactory that Bob does not kill himself on the original timeline, I believe that the "branching solution" or multiple timelines view of time travel is the only way any sense can be made of time travel; and it is the meaning of time travel portrayed in most fictional accounts (AFAIK).

    It should be noted that the singular timeline view of time travel, such as that depicted in the Grandfather paradox, does not permit the time traveller to kill themselves on the original timeline, either, because this is what produces paradoxical results. The time traveller cannot visit an earlier time on the original timeline without falling into contradiction, because there was either no time traveller at the destination time "the first time around" or because, otherwise, the time traveller was always at the destination time (i.e. the first time around) and time travel is therefore impossible (or, at least, unnecessary). The desire to time travel in order to alter the history of the original timeline that leads to the point at which the time traveller departs in their time machine is therefore futile on a singular timeline view. The time traveller was either never at the destination time and cannot return there without contradiction (having two conflicting histories on a single timeline), or else they were always there and therefore cannot "return" there.

    On the multiple timeline view, although Bob creates a second timeline as a result of his time travel, at least from his perspective, it is possible for him to return to the past to do whatever he likes, including killing his younger self. Moreover, for time travel to make any sense, Bob must travel to, and insert himself into, a past time at which he didn't always already exist as a time traveller.

    I believe the multiple timeline view is what is usually understood to be time travel and is the only way time travel makes any sense. Understandably, however, it may not satisfy those who want to alter the original timeline or murder any of its past inhabitants.
  • noAxioms
    1.4k
    The time traveller was either never at the destination time and cannot return there without contradiction (having two conflicting histories on a single timeline), or else they were always there and therefore cannot "return" there.Luke
    How is the 2nd clause different from the first? Both just seem to say that you can't travel to your own timeline, which is partly silly because I am doing it now. IOW, does forward time travel necessitate a branch in timelines?

    One can pretty easily do it just by having a space ship that travels faster than light, and fiction is full of that as well.

    Bob must travel to, and insert himself into, a past time at which he didn't always already exist as a time traveller.
    Why this restriction? I go back to 1955 (standard destination). Hang around until 1970, and go back to 1960 this time, where "I" already am as a time traveler. What's wrong with that? Can he also make a 3rd branch off the original timeline? Can I, having just made the machine, branch a new line off some other timeline where I never existed in the first place, say some version of 1980 where my parents didn't survive WWII?

    Meanwhile, why do you want to kill anybody? The young-Luke you find back there is not you since 'you' is presumably on the original timeline. You've no reason to kill this other person or for that matter, anybody. If you kill yourself, have you killed Luke, or did a copy kill himself?
    I think you need to consider the question I asked about the Studebaker in my first post in this thread (about post 57). Is that time travel? If not, why not? What is your machine doing that my example with the Studebaker did not?
  • AmadeusD
    2k
    or murder any of its past inhabitants.Luke

    :lol: I like your faith in humanity, Luke.

    does forward time travel necessitate a branch in timelines?noAxioms

    Surely, this is now talking about a "possible future" that you've gone in to - it may not be the actual future of your OG time-line. You being in the future isn't a contradiction here, though, as you're not in the OG timeframe. You just disappeared for a while in the OG one and reappeared at your 'destination' moment in time.
    Then, the 'branch' would happen, I would suppose at a point in the OG timeframe that contradicted the future branch you've travelled to. This might create an alternate past, pro-actively (as far back as your moment of travel in the OG timeline) and I haven't considered if that's an issue. It seems unavoidable that there would be a branch, but not logically necessary. Very much happy to have this ripped apart though as i'ts taken me 10 minutes between settlements at work.

    Can I, having just made the machine, branch a new line off some other timeline where I never existed in the first place, say some version of 1980 where my parents didn't survive WWII?noAxioms

    Surely this would just create a branch into which you've traveled regardless, leaving the OG timeline without you, or your parents, in it? A timeline into which you traveled and now exist rather than existing prior to the travel.
  • jgill
    3.6k
    For all the discussions about the Grandfather Paradox, timelines, alternate histories, etc. I don't think humans will ever travel physically to the past, although in theory we are now able to travel to the future slower and slower via spaceships.

    We "travel" to the past when we look at an old photo or watch an old TV show. That kind of technology might develop into a means of recording virtually everything in a particular sphere of activity, allowing those in the future to "travel" into the past without any paradoxes or violations of physical laws. In this sense time travel is dependent upon the present preparing for the future travelers.

    If you think physical time travel to the past is possible, begin by explaining how.
  • Luke
    2.6k
    The time traveller was either never at the destination time and cannot return there without contradiction (having two conflicting histories on a single timeline), or else they were always there and therefore cannot "return" there.
    — Luke

    How is the 2nd clause different from the first?
    noAxioms

    According to the second clause, the time traveller always appears alongside their younger self in the past. The problem with this view is that the effects of time travel precede the causes of time travel. Given the normal temporal order of things from past to future, Bob appears in the past alongside his younger self in this scenario before he has built a time machine or time travelled. Therefore, there is no longer any need for Bob to continue on to build a time machine. We may ask where is the starting point of this causation chain? There exists a paradoxical temporal loop in which causes are effects and effects causes. Presumably, Bob built the time machine (in his later years) for the purpose of returning to the past, so did he forget that he was already in the past? Where do time traveller Bob's memories start and end in this scenario? If time traveller Bob is always in the past, then how can he return to the past via time travel?

    The first clause presumes that time traveller Bob is not already in the past ("the first time around") before he has time travelled. However, if Bob were to subsequently time travel to the past, then this would create two different histories on the same timeline; one in which time traveller Bob is not in the past and another in which time traveller Bob is in the past.

    Both just seem to say that you can't travel to your own timeline, which is partly silly because I am doing it now.noAxioms

    I wouldn't call that time travel in the relevant sense. The SEP article attempts to draw the relevant distinction: https://plato.stanford.edu/entries/time-travel/#WhaTimTra

    IOW, does forward time travel necessitate a branch in timelines?noAxioms

    I think forward time travel is a bit trickier, but I suspect that it does also require a branch in timelines. I would prefer to consider the "simpler" backwards time travel for now.

    Bob must travel to, and insert himself into, a past time at which he didn't always already exist as a time traveller.
    — Luke

    Why this restriction? I go back to 1955 (standard destination). Hang around until 1970, and go back to 1960 this time, where "I" already am as a time traveler. What's wrong with that? Can he also make a 3rd branch off the original timeline?
    noAxioms

    My point was that it is senseless for Bob to travel to the past if he is already there (as is the case with a single eternalist timeline). I don't foresee issues with additional branching. I was only considering a singular time travel event.

    Can I, having just made the machine, branch a new line off some other timeline where I never existed in the first place, say some version of 1980 where my parents didn't survive WWII?noAxioms

    I don't think so. I'm assuming that Bob returns to the same past that he lived through when he was younger. It's just that he cannot have been there as a time traveller the first time around, before he built his time machine.

    Meanwhile, why do you want to kill anybody?noAxioms

    I don't; that's the scenario of the Grandfather paradox. For example, see here: https://plato.stanford.edu/entries/time-travel/#GraPar

    The young-Luke you find back there is not you since 'you' is presumably on the original timeline. You've no reason to kill this other person or for that matter, anybody. If you kill yourself, have you killed Luke, or did a copy kill himself?noAxioms

    Call it a copy if you will but this is the only way that time travel is possible. It is not possible for Bob to return to the past on the original timeline due to the resulting violations of causality and contradictions. The second timeline differs from the original only in that there is no time traveller on the original timeline until after Bob has constructed and used his time machine. The only way for Bob to return to the past to meet his younger self is on a second timeline, so the only way for Bob to return to the past and kill his younger self is on a second timeline.

    If you kill yourself, have you killed Luke, or did a copy kill himself?noAxioms

    I assume that it is the Bob (or Luke) from the original timeline who time travels to the past and creates a second timeline in the process. Bob therefore remains on the original timeline until his time travel event when he relocates to the second timeline.

    I think you need to consider the question I asked about the Studebaker in my first post in this thread (about post 57). Is that time travel? If not, why not? What is your machine doing that my example with the Studebaker did not?noAxioms

    As I replied earlier, I wouldn't call this recreation of another time in the present time to be time travel. I am assuming that Bob's time machine actually works and transports him to a period of time other than the (present) time that we are currently living through together.
  • noAxioms
    1.4k
    I wouldn't call that time travel in the relevant sense. The SEP article attempts to draw the relevant distinction: https://plato.stanford.edu/entries/time-travel/#WhaTimTraLuke
    I read almost all of the SEP article. Thanks for the link. Didn't know it had a page on the subject.
    Apparently what I am doing right now does count as time travel, so long as I move. Motion-related time dilation apparently counts according to the article. I would not have said that.

    The definition they referenced (but were not totally satisfied with) came from Lewis (1976) who says that time travel is when two different kinds of time (coordinate time and proper time say) don't match. The clock on the wall says one thing, but my watch says another. But physics allows that sort of stuff all the time. One is always proper time, but the other is something else. The SEP article seems not to know physics enough to actually use the correct term of 'proper time', the length (interval) of any worldline, which is what a clock measures if it follows that worldline.

    The article doesn't ever reference 'branching', but it talks about traveling to other parallel timelines, like those lines already existed, and apparently already have the time machine appearing from nothing in them, much to the surprise of any witnesses.

    The article does seem to accept travel to a 'nonexistent' time in the past under presentism. All of section 4 discusses the various interpretations of time and their implications.

    My point was that it is senseless for Bob to travel to the past if he is already thereLuke
    Him already being there was the point: To alter what he (younger self) would have otherwise done. I see no reason why the younger self cannot have already time travelled before. Another mistake could be made, 'necessitating' a second correction. I put it in quotes because the mistake cannot be corrected on the more original (more real?) timeline.

    I don't think so. I'm assuming that Bob returns to the same past that he lived through when he was youngerLuke
    Poor assumption. If I'm to 'kill grandfather', I'd have to go back at least a century. Maybe I want to witness the asteroid taking out the dinosaurs. You can't put in a rule that says you can only travel a short ways to some past with you in it somewhere.

    You also contradict yourself. You say on one hand that it is senseless to go to a time when you exist, and on the other hand you're presuming Bob does this 'senseless' thing.

    I don't; that's the scenario of the Grandfather paradox.Luke
    Any travel to the distant past will destroy the history you know. Everyone talks about critical events that make a change, but just appearing and stepping on a bug is enough. That said, killing grandpa isn't necessarily paradoxical. Maybe you're not actually related to him, but rather the mailman. I know my grandfather was a cheater. Why can't grandma be?

    Call it a copy if you will but this is the only way that time travel is possible.Luke
    Well, besides the fact that it isn't possible in the first place, there are valid scenarios discussed in SEP that allow travel to the original timeline. CTCs are one example.

    I assume that it is the Bob (or Luke) from the original timelineLuke
    I don't make that assumption. I try to work it out.

    As I replied earlier, I wouldn't call this recreation of another time in the present time to be time travel.Luke
    I'm not recreating a time. I'm just moving a Studebaker forward in time by a century. That's pretty much exactly what you're describing, except in the forward direction. So tell me why that's not what I did. How do you back the assertion that the car didn't travel through time, but Bob (also going forward say) did. Was it the lack of a fancy time machine looking device with blinking lights and stuff? There's plenty of fictions where the machine isn't necessary.

    They've done quantum teleportation, which counts as time travel according to SEP. They put something in a booth and it teleported it to another a couple hundred km away. Arguably not time travel, but my question is: Is the thing at the far booth (and no longer at the near one) the same object, or a perfect copy? They were asked this question, and replied: "What possible difference does it make?".
  • Luke
    2.6k
    Apparently what I am doing right now does count as time travel, so long as I move.noAxioms

    As long as you move at a relatively different speed to the rest of us, I suppose. Anyhow, that's not quite what you said earlier, which was that you are time travelling by staying on your own timeline. This could be achieved by something like 'waiting', which the SEP article categorises as not time travel.

    My point was that it is senseless for Bob to travel to the past if he is already there
    — Luke

    Him already being there was the point: To alter what he (younger self) would have otherwise done.
    noAxioms

    I agree that the point of the time travel is for him to be at the earlier time in order to alter what he would have otherwise done. But this is what necessitates his time travel to the earlier time destination. There is no point or possibility of travelling to a destination if you are already there. That is my point.

    I see no reason why the younger self cannot have already time travelled before.noAxioms

    The younger self does not time travel; the older self does. Old Bob travels back to the time when he was younger. In the context of the Grandfather paradox (or one of its variants), the purpose of the time travel is for Old Bob (the time traveller) to murder his younger self. The younger self has not yet grown up to build a time machine, so he cannot have already time travelled before. Obviously, there may be scenarios - unrelated to the Grandfather paradox - in which Young Bob jumps into Old Bob's time machine after he arrives and then Young Bob uses the time machine to time travel to another time, but I see no point in complicating matters further.

    Another mistake could be made, 'necessitating' a second correction. I put it in quotes because the mistake cannot be corrected on the more original (more real?) timeline.noAxioms

    I haven't suggested that the original timeline is "more real" than others. However, you have suggested that the inhabitants of the second timeline are merely "copies".

    I don't think so. I'm assuming that Bob returns to the same past that he lived through when he was younger
    — Luke

    Poor assumption. If I'm to 'kill grandfather', I'd have to go back at least a century. Maybe I want to witness the asteroid taking out the dinosaurs. You can't put in a rule that says you can only travel a short ways to some past with you in it somewhere.
    noAxioms

    I don't rule out that Bob can travel to some time before his birth. I was referring to the context of the (Grandfather paradox) scenario I've described, where Old Bob time travels in order to murder his younger self.

    You also contradict yourself. You say on one hand that it is senseless to go to a time when you exist, and on the other hand you're presuming Bob does this 'senseless' thing.noAxioms

    I'm saying time travel is senseless on a single timeline, and that it only makes sense if there is more than one timeline; where a second (or more) timeline is created as a result of the time travel event.

    Any travel to the distant past will destroy the history you know. Everyone talks about critical events that make a change, but just appearing and stepping on a bug is enough. That said, killing grandpa isn't necessarily paradoxical. Maybe you're not actually related to him, but rather the mailman. I know my grandfather was a cheater. Why can't grandma be?noAxioms

    For the sake of argument, let's presume that he really is your grandfather. Then it would be paradoxical; at least, on a single timeline.

    Well, besides the fact that it isn't possible in the first place, there are valid scenarios discussed in SEP that allow travel to the original timeline. CTCs are one example.noAxioms

    Again, I was speaking within the context and logic of the Grandfather paradox, which is presumably a 'doctor' or 'leap' type of time travel rather than the others described in the article. I wasn't really considering those other sorts of time travel scenarios.

    I assume that it is the Bob (or Luke) from the original timeline
    — Luke

    I don't make that assumption. I try to work it out.
    noAxioms

    You asked whether it was Old Bob or only a copy of Old Bob who killed his younger self. My answer was that it is Old Bob from the original timeline who time travels and kills his younger self (on the new timeline). When Old Bob time travels, he disappears from the original timeline (at his time of departure) and appears on the second timeline (at his time of arrival).

    As I replied earlier, I wouldn't call this recreation of another time in the present time to be time travel.
    — Luke

    I'm not recreating a time. I'm just moving a Studebaker forward in time by a century. That's pretty much exactly what you're describing, except in the forward direction. So tell me why that's not what I did. How do you back the assertion that the car didn't travel through time, but Bob (also going forward say) did. Was it the lack of a fancy time machine looking device with blinking lights and stuff?
    noAxioms

    Going by your earlier description:

    Recreating a piece of some past state. Indeed, this isn't time travel being described.
    I can build a new 1928 Studebaker, even giving it the same serial number as one made in that year. Has that car time traveled or is it just a new thing? I satisfied the conditions of the OP by doing so. Is it even a Studebaker if I built it instead of the defunct company?
    noAxioms

    I've said that I'm not (or that Bob is not) recreating a past time (e.g. 1990) in the present, and that Bob actually time travels to 1990. You say that recreating the 1928 Studebaker now is not time travel and I say that recreating the year 1990 now is not time travel. If you were to recreate it today, then you haven't really moved a Studebaker forward in time by a century. As you note, "this isn't time travel being described". I thought we were on the same page here?
  • noAxioms
    1.4k
    This could be achieved by something like 'waiting', which the SEP article categorises as not time travel.Luke
    The SEP does consider travel to a parallel timeline (Meiland, 1974 or Deutsch and Lockwood, 1994) to be time travel despite the lack of rigorous simultaneity convention between separate timelines. Your idea seems to illustrate the same issue. What was this alternate timeline doing before Bob traveled to it? Was it different in any way? Did it have a 'present' 2024 that was altered by Bob's appearance in what was considered to be 1990 at the time?
    I don't think your timelines are parallel like the ones discussed in SEP. The question seems more approriate for the above mentioned authors.

    However, you have suggested that the inhabitants of the second timeline are merely "copies".Luke
    You seem to regard them as copies yourself, as evidenced by several comments (my bold):

    There is no point or possibility of travelling to a destination if you are already there.
    So by this wording, the young Bob that gets killed is not Bob. He is not already there, but is rather killing a copy, somebody else, having left the young Bob that is actually himself back in the original timeline unkilled.

    The younger self does not time travel; the older self does.
    If the two of them were the same person, this would be a direct contradiction. But you seem to regard them as not the same person. So if (actual) Bob goes to some parallel world in 1990, and waits several years for the perfect opportunity to take out the young-Bob copy1 that is there. The moment comes, and he fires his gun only to find it wasn't loaded. Opportunity lost, and there won't be another one. But he has a time machine, so he goes back a day and loads the gun that yesterday-Bob (also a copy) can use to complete his task (of killing young-Bob copy2, leaving young-Bob copy1 un-shot back in the first alternate timeline).

    My point of all that is that your comment is true only if you assume yesterday-Bob is a copy. Yesterday-Bob (the one with the empty gun) has time traveled (he's the original Bob). Yesterday-Bob copy has not, him being a different person with a false memory of having time traveled. If he was the same person as Bob, then he very much as time traveled, explaining his presence in 1995 and his memory of 2024.

    I'm saying time travel is senseless on a single timelineLuke
    All the examples of 'is time travel' at the top of the SEP article are single-timeline examples. I'm not saying that traveling 'sideways' to a different line is or is not time travel, but I'm saying that those examples cannot all be senseless. Yes, they all have potential paradoxical consequences, all discussed in the article.

    My answer was that it is Old Bob from the original timeline who time travels and kills his younger self (on the new timeline).Luke
    My reason for asking was to figure out justification of that assertion. I'm not saying it's wrong, just an arbitrary designation. Most designations of identity have pragmatic reasoning and are thus not arbitrary. This doesn't, so the question needs asking, and the answer needs justification.
    You wanted to explore the implications. I'm trying to do that.
  • Luke
    2.6k
    What was this alternate timeline doing before Bob traveled to it?noAxioms

    It did not exist before Bob travelled to it; the new timeline is created by Bob's time travel. It is the same as the original timeline, except now with the addition of a time traveller: Bob.

    Did it have a 'present' 2024 that was altered by Bob's appearance in what was considered to be 1990 at the time?noAxioms

    The new timeline did not exist before Bob arrived, so there was no "original" version of the new timeline to alter. The original version is the original timeline, from which Bob departs. The original timeline does not have a time traveller arriving in its 1990. 1990 on the original timeline contains only young Bob (sans time traveller) who grows up to be time-travelling Old Bob. It is only on the new timeline that young Bob meets time-travelling Old Bob; when Young Bob and Old Bob exist together at the same time. The new timeline is created when Bob travels from the future (of the original timeline). To illustrate:

    Original timeline:
    1990: Young Bob (only) who grows up to become Old Bob the time traveller
    2024: Old Bob time travels to 1990 in order to kill Young Bob.
    2024 (post-time travel): Bob is no longer on this (original) timeline from this time onwards.

    New timeline:
    1990: Old Bob arrives in 1990. The new timeline now contains Young Bob and Old Bob together at the same time (this is not the case on the original timeline). Old Bob kills Young Bob.
    1990 (post Young Bob's murder): Old Bob lives out his life.

    If Old Bob was at 1990 on the original timeline (i.e. on a single timeline), then the time travel event must have occurred before Bob built the time machine.

    You seem to regard them as copies yourself, as evidenced by several comments (my bold):noAxioms

    You can consider them to be copies, I suppose, but it's the only way I can make sense of the occurrence of time travel. It's not a copy of Old Bob, since he time travels from the original timeline to the new timeline. But the new timeline might be considered to contain a copy of everything/everyone else. Alternatively, the new timeline could be considered as how the (original) past gets changed by the presence of a time traveller from the future (following the time of their arrival). I believe that is what we should expect from a time travel event. If you are going to time travel back in time to kill Hitler as a baby, then Hitler can't have already been killed as a baby in the past. What's the point (or possibility) of time travelling to the past if it is to leave the past completely unchanged?

    There is no point or possibility of travelling to a destination if you are already there.
    — Luke

    So by this wording, the young Bob that gets killed is not Bob. He is not already there, but is rather killing a copy, somebody else, having left the young Bob that is actually himself back in the original timeline unkilled.
    noAxioms

    It is (Young) Bob that gets killed, but not (Young) Bob from the original timeline, that's true. Young Bob from the original timeline is the one that grows up to be Old Bob the time traveller/murderer. This is the only way to avoid contradictions, paradox and violations of causality.

    The younger self does not time travel; the older self does.
    — Luke

    If the two of them were the same person, this would be a direct contradiction. But you seem to regard them as not the same person.
    noAxioms

    I do regard them as the same person at different times, but the logic of time travel allows for an older version of a person to meet their younger self. That is the scenario of the Grandfather paradox.

    So if (actual) Bob goes to some parallel world in 1990, and waits several years for the perfect opportunity to take out the young-Bob copy1 that is there. The moment comes, and he fires his gun only to find it wasn't loaded. Opportunity lost, and there won't be another one. But he has a time machine, so he goes back a day and loads the gun that yesterday-Bob (also a copy) can use to complete his task (of killing young-Bob copy2, leaving young-Bob copy1 un-shot back in the first alternate timeline).noAxioms

    You could consider them to be copies or you could consider it to be Bob actually changing the past from what it was originally. Obviously, Bob cannot change the past on the original timeline, because he was not in the past (as a time traveller) on the original timeline. If Bob were in the past (as a time traveller) on the original timeline, then his time travel would not change the past. In fact, in that case, Old Bob would be in the past before his time travel event (or without needing to time travel). It is only if Bob were not in the past (as a time traveller) on the original timeline that he could change the past from what it was (albeit on a new timeline) by the mere insertion of himself into the past as a time traveller.

    Although Bob cannot change the past of the original timeline, he can create a new and different past to that of the original timeline, which I think is as much as can be expected while also avoiding paradox or contradiction. In order to change the past, the past needs to have been a particular way previously. Similarly, if I want to change my hair colour from brown to blue, I can't change the fact that it was brown originally.

    All the examples of 'is time travel' at the top of the SEP article are single-timeline examples. I'm not saying that traveling 'sideways' to a different line is or is not time travel, but I'm saying that those examples cannot all be senseless. Yes, they all have potential paradoxical consequences, all discussed in the article.noAxioms

    Right, but I'm attempting to point out why I think single-timeline examples of time travel are senseless, and why I believe that a second timeline is necessary to avoid contradiction or paradox. However, I may have done a very poor job of that so far.

    My answer was that it is Old Bob from the original timeline who time travels and kills his younger self (on the new timeline). — Luke

    My reason for asking was to figure out justification of that assertion. I'm not saying it's wrong, just an arbitrary designation. Most designations of identity have pragmatic reasoning and are thus not arbitrary. This doesn't, so the question needs asking, and the answer needs justification.
    noAxioms

    I am arguing that Old Bob cannot have been in the past originally, because Young Bob had not yet grown up to build a time machine or to time travel.

    You wanted to explore the implications. I'm trying to do that.noAxioms

    I appreciate it. Hopefully I've helped clarify my position a little better with this post.
  • noAxioms
    1.4k
    It's not a copy of Old Bob, since he time travels from the original timeline to the new timeline.Luke
    That part is just you saying it. It could be just as easily said that everybody (including old Bob) in the copy timeline is a copy. The machine could split Bob just like it splits everything else. The story doesn't go like that, but the story could go like that. It would still be time travel of a sort, especially from the PoV of the Bob on the created timeline.

    What's the point (or possibility) of time travelling to the past if it is to leave the past completely unchanged?
    I take it you're not a historian. Those guys would love a machine that lets them go back, even in a way that cannot alter anything, just watch.

    This is the only way to avoid contradictions, paradox and violations of causality.
    I think we're doing considerable damage to causality if any of this were plausible. OK, the Einstein time travel doesn't violate causality, but I personally don't think that one counts even if it meets the SEP definition.

    Right, but I'm attempting to point out why I think single-timeline examples of time travel are senseless, and why I believe that a second timeline is necessary to avoid contradiction or paradox.
    The SEP article gives several examples of a single timeline without paradox, Some of the best are the loop ones, including a case where you don't even need to invent/build the machine. You just give it to your younger self when you're done with it.
    There is a sort of paradox with that scenario which is how the machine experiences no entropy: It stays perfectly new at all times, which isn't plausible for something that is thousands of years old.

    Most designations of identity have pragmatic reasoning and are thus not arbitrary. This doesn't, so the question needs asking, and the answer needs justification.
    — noAxioms

    I am arguing that Old Bob cannot have been in the past originally, because Young Bob had not yet grown up to build a time machine or to time travel.
    Luke
    You don't seem to understand my point, which is that there is not obvious convention as to if the old-Bob in the copy timeline is the same old-Bob from the original timeline. The usual conventions for saying this person is the same person that looked like him yesterday. "I bought a can of beans yesterday": True? By convention, yes, the person who bought the can of beans is the same person that submitted this post. We know that because we know the convention. There is no convention for crossing timelines. To me it looks like old-Bob commits suicide, but builds a copy of himself (and the machine) in a timeline with a copy of everything else. The convention could just as easily say that.
  • Luke
    2.6k
    It's not a copy of Old Bob, since he time travels from the original timeline to the new timeline. — Luke

    That part is just you saying it. It could be just as easily said that everybody (including old Bob) in the copy timeline is a copy. The machine could split Bob just like it splits everything else. The story doesn't go like that, but the story could go like that. It would still be time travel of a sort, especially from the PoV of the Bob on the created timeline.
    noAxioms

    That could be, but the existence of Bob and his time machine ends on the original timeline with the time travel event. This does not happen to anything else on the original timeline.

    Also, Bob has built the time machine with the intention of returning to the past, so he should not be surprised by his sudden appearance at an earlier time, unlike everyone else on the new timeline (who we would assume have never encountered a time traveller before).

    Also, instead of considering the new timeline as a copy, you could consider it as a re-writing of history, but one which does not eliminate the original timeline.

    What's the point (or possibility) of time travelling to the past if it is to leave the past completely unchanged?

    I take it you're not a historian. Those guys would love a machine that lets them go back, even in a way that cannot alter anything, just watch.
    noAxioms

    Ah, but if they were to travel to the past then that would be altering something about the past; namely, that the historians were not originally in the past (as time travellers). Their additional existence in the past, even as mere spectators, would change the past. This is the most basic contradiction in relation to time travel on a single timeline.

    I think we're doing considerable damage to causality if any of this were plausible. OK, the Einstein time travel doesn't violate causality, but I personally don't think that one counts even if it meets the SEP definition.noAxioms

    It's not the sort of time travel I had in mind, either.

    Incidentally, based on my very amateur understanding, I had thought that once the Einsteinian "time traveller" had returned to Earth, the same amount of time must have elapsed on Earth as it has for the traveller, given the time dilation effects of turning their ship around in order to return. When I read about the twin paradox long ago, I figured that although one twin can be in the future of the other, there is no way to transmit information to the Earthbound twin which could give them advanced knowledge about the future and that they must both return to the same proper time when they meet again. However, I admit that I don't fully understand these things and I'm probably way off. Besides, those sorts of time travel scenarios involving that type of "time travel" are not what I had in mind here.

    The SEP article gives several examples of a single timeline without paradox, Some of the best are the loop ones, including a case where you don't even need to invent/build the machine. You just give it to your younger self when you're done with it.noAxioms

    There are three scenarios described in section 3.2 on causal loops. I reject them on the same basis as the arguments I have given here. I will respond to each of them below.

    ...imagine a time traveller who steals a time machine from the local museum in order to make his time trip and then donates the time machine to the same museum at the end of the trip (i.e. in the past). In this case the machine itself is never built by anyone—it simply exists.

    The scenario wants us to imagine that this is a logically-sealed causal loop. However, the time machine must have been built by someone else in order for it to have been stolen and then donated to the museum. It wasn't the time traveller that built it, so it cannot be the donation by the time traveller that causes the existence of the time machine.

    The logical order of things is that:
    (i) someone else built the time machine;
    (ii) the time machine ends up in the museum (somehow other than via donation by the time traveller);
    (iii) the time traveller steals the time machine from the museum;
    (iv) the time traveller use the time machine to return to the past;
    (v) the time traveller donates the time machine to the museum.

    The causal loop (steps (iii)-(v)) begins only after the time travel event, but it cannot logically eliminate steps (i) and (ii). Logically, someone must have built the time machine to begin with; such things cannot "simply exist" uncaused. This is much like what I am suggesting with the original timeline of Bob. Young Bob first has to grow up through a period without time travellers in order to build the time machine and return to the past as Old Bob.

    The time traveller must have possession of their time machine before the causal loop begins, in a way which does not rely on the causal loop. Otherwise, the time traveller cannot time travel in order to create the causal loop. If you don't have a time machine then you can't time travel, so you can't then obtain that time machine (that you don't have) via time travel (that you can't do).

    ...imagine a time traveller who explains the theory behind time travel to her younger self: theory that she herself knows only because it was explained to her in her youth by her time travelling older self.

    The time travelling older self must have had this knowledge before the first time travel event, and must have obtained this knowledge in some way other than via the time travel event. Without the knowledge to begin with, they could not have time travelled and therefore could not have given this information to their younger self. They must have obtained this knowledge by some other means prior to first time travelling. This is another attempt to eliminate the start/cause of the causal loop.

    Imagine a time traveller who visits his younger self. When he encounters his younger self, he suddenly has a vivid memory of being punched on the nose by a strange visitor. He realises that this is that very encounter—and resignedly proceeds to punch his younger self. Why did he do it? Because he knew that it would happen and so felt that he had to do it—but he only knew it would happen because he in fact did it.

    I don't buy the fact that the time traveller could not have done something else. Their only constraint is the presumption of a single timeline in this scenario and the avoidance of contradiction.

    There is a sort of paradox with that scenario which is how the machine experiences no entropy: It stays perfectly new at all times, which isn't plausible for something that is thousands of years old.noAxioms

    I don't follow why it must be thousands of years old. I imagine the causal loop in these scenarios to be a much shorter period than thousands of years.

    I am arguing that Old Bob cannot have been in the past originally, because Young Bob had not yet grown up to build a time machine or to time travel.
    — Luke

    You don't seem to understand my point, which is that there is not obvious convention as to if the old-Bob in the copy timeline is the same old-Bob from the original timeline. The usual conventions for saying this person is the same person that looked like him yesterday. "I bought a can of beans yesterday": True? By convention, yes, the person who bought the can of beans is the same person that submitted this post. We know that because we know the convention. There is no convention for crossing timelines. To me it looks like old-Bob commits suicide, but builds a copy of himself (and the machine) in a timeline with a copy of everything else. The convention could just as easily say that.
    noAxioms

    If we don't conventionally say that we are mere copies of ourselves in our normal passage through time from moment to moment, then I see no reason to apply a different convention to time travellers.
  • noAxioms
    1.4k
    he should not be surprised by his sudden appearance at an earlier time, unlike everyone else on the new timeline (who we would assume have never encountered a time traveller before).Luke
    By your rules, a person can only be in the presence of but one actual time traveler, even if other people on the timeline also remember time traveling.
    Any witnesses to Bob's first moment's on that timeline will be surprised by the apparent sudden appearance of Bob, despite the fact that he has been there all along and by definition doesn't appear somewhere where he wasn't just before. There simply isn't a 'just before' on that timeline.

    Also, instead of considering the new timeline as a copy, you could consider it as a re-writing of history, but one which does not eliminate the original timeline.
    That would be a different convention. The new timeline is a rewrote-history according to traveled-Bob, and the old timeline becomes the copy from which he originated.

    if they were to travel to the past then that would be altering something about the past
    There are stories/scenarios in which nothing is altered. It's more like watching the past on TV since nothing there can detect you.

    Incidentally, based on my very amateur understanding, I had thought that once the Einsteinian "time traveller" had returned to Earth, the same amount of time must have elapsed on Earth as it has for the traveller, given the time dilation effects of turning their ship around in order to return. When I read about the twin paradox long ago, I figured that although one twin can be in the future of the other, there is no way to transmit information to the Earthbound twin which could give them advanced knowledge about the future and that they must both return to the same proper time when they meet again. However, I admit that I don't fully understand these things and I'm probably way off. Besides, those sorts of time travel scenarios involving that type of "time travel" are not what I had in mind here.
    The twin goes out and comes back, and the two twins are no longer the same age. Not sure what you've been reading, but the proper time going out and back is less than the proper time of a direct path between the two events where the depart and meet up again. None of this requires anything communicating or having knowledge of what the other is doing.
    It works better with pregnant women, who make great clocks. Betty and Veronica both get knocked up the same day and Betty takes off for the stars. Betty comes back in 9 months her time and has her baby. She meets who she presumes to be Veronica also giving birth in the same hospital, but it turns out the woman is actually Veronica's daughter giving birth to Veronica's grandchild. 20 some years have passed on Earth since Betty left and the other woman is merely the spitting image of her mother.

    Anyway, SEP considers that to be time travel.

    The scenario wants us to imagine that this is a logically-sealed causal loop. However, the time machine must have been built by someone else in order for it to have been stolen and then donated to the museum.
    Does not follow. That sort of reasoning is only valid if time travel is not possible. The whole point is that it was never built.

    It wasn't the time traveller that built it, so it cannot be the donation by the time traveller that causes the existence of the time machine.
    The existence is caused by its own time travel to the past. Such is the nature of closed loops. Still, in my prior post I pointed out a hole in that story.

    ...imagine a time traveller who explains the theory behind time travel to her younger self: theory that she herself knows only because it was explained to her in her youth by her time travelling older self.
    That version works better since it mostly solves the problem I identified.

    I don't buy the fact that the time traveller could not have done something else.
    The closed-loop scenarios illustrate free will (or more precisely, the lack of it) better than any discussion about reality where there's no pragmatism to it.

    There are several fiction stories with closed loops. Predestination is one of them.

    [The machine] stays perfectly new at all times, which isn't plausible for something that is thousands of years old.
    — noAxioms

    I don't follow why it must be thousands of years old.
    It takes say 10 years from donation to museum to getting stolen. It ages 10 years during that time after which it goes back 10 years and does it again, and again... Infinite age since it's stuck in a loop. Somebody has to do one excellent refurbishment effort somewhere during each 10 years.

    I imagine the causal loop in these scenarios to be a much shorter period than thousands of years.
    It's a loop. It has no finite length, just a period, just like there's no end to walking east.
  • Luke
    2.6k
    he should not be surprised by his sudden appearance at an earlier time, unlike everyone else on the new timeline (who we would assume have never encountered a time traveller before).
    — Luke

    By your rules, a person can only be in the presence of but one actual time traveler, even if other people on the timeline also remember time traveling.
    noAxioms

    I'm considering Bob to be the first ever time traveller. This is because I find the logic of single timelines and closed loops to be problematic at the first time travel event. Given that Bob creates the first time travel event, nobody has ever encountered a time traveller before.

    Also, instead of considering the new timeline as a copy, you could consider it as a re-writing of history, but one which does not eliminate the original timeline.
    — Luke

    That would be a different convention. The new timeline is a rewrote-history according to traveled-Bob, and the old timeline becomes the copy from which he originated.
    noAxioms

    I don't disagree with this, except for your insistence that we call it a copy. In a sense it is, but it is also the only way that time travel can work; the only way a time traveller can visit a past they've never visited before without causing a contradiction, If you change the past from being history A to being history B, then you can't eliminate the fact that it was history A before you made it history B..The different histories are the different timelines. Otherwise, on a single timeline, there would be a contradiction: the past both does and does not contain a time traveller.

    if they were to travel to the past then that would be altering something about the past
    — Luke

    There are stories/scenarios in which nothing is altered. It's more like watching the past on TV since nothing there can detect you.
    noAxioms

    Well, I wouldn't call that time travel or "travelling to the past", That is just somehow viewing the past at the present time of the viewers.

    The scenario wants us to imagine that this is a logically-sealed causal loop. However, the time machine must have been built by someone else in order for it to have been stolen and then donated to the museum.
    — Luke

    Does not follow. That sort of reasoning is only valid if time travel is not possible. The whole point is that it was never built.
    noAxioms

    I understand the intended point of the example. You will need to explain why my objection does not follow. The "logical order of things" I described included time travel, so I don't see why my reasoning is valid only if time travel is not possible.

    It wasn't the time traveller that built it, so it cannot be the donation by the time traveller that causes the existence of the time machine.
    — Luke

    The existence is caused by its own time travel to the past. Such is the nature of closed loops.
    noAxioms

    I understand that is the intended point of the scenario. However, I raised an objection to its logic, which was:

    "If you don't have a time machine then you can't time travel, so you can't then obtain that time machine (or its technology) via time travel."

    The examples provided rely on the time traveller having obtained their time machine or its technology via time travel before they ever had the means to time travel.

    Since you can't time travel without a time machine, it cannot be a closed loop. The chain of causes cannot begin after the time travel event, because there can be no time travel without a time machine. Being in possession of a time machine must be the initial cause of everything that follows, so the initial possession of a time machine cannot be the effect of its time travel event.

    Still, in my prior post I pointed out a hole in that story.noAxioms

    Which hole are you referring to? Entropy?

    Infinite age since it's stuck in a loop. Somebody has to do one excellent refurbishment effort somewhere during each 10 years.noAxioms

    The loop could have started only 10 years ago. Also, the advanced technology of a time machine could give it thousands of years of repair-free use, but I see your point.
  • noAxioms
    1.4k
    By your rules, a person can only be in the presence of but one actual time traveler, even if other people on the timeline also remember time traveling.
    — noAxioms

    I'm considering Bob to be the first ever time traveller..
    Luke
    You're not reading my comment. I said that by your rules, a person can be in the presence of at most one actual time traveler. We could have a factory that made them like bags of cheetos, and everybody used them to get to appointments and catch the traffic light that just went yellow. If they were used like that, the planet would quickly have a population of zero in not just the original, but all the timelines. Despite that prediction, no person would ever be in the presence of more than one actual time traveler, which is the one and only person that created the specific timeline the person finds himself in (if he's still in it and hasn't left already).
    Actually, nobody would use the machines, due to the overwhelming evidence of it being nothing more than a self-annihilation machine. So good thing Bob is the only person that has one, and only Bob fails to exist in pretty much any of the timelines.

    if they were to travel to the past then that would be altering something about the past.
    But the way you describe it, it isn't really the past, just a different timeline which maybe looks like 'the' past, but is actually just another line, 'a' past at best, one of many. There is only one 'the' past, and you didn't go there.

    Well, I wouldn't call [a read-only verision] "travelling to the past", That is just somehow viewing the past at the present time of the viewers.
    Except he can't leave (turn off the TV so to speak). OK, I agree that it stretches the definition too much. But if he's there at all, history is gone. If I go back 250 million years to see the early evolution of mammals, I'm sorry, but humans will never evolve from that timeline. Your very presence destroys that, although it doesn't prevent the asteroid that wipes out whatever is there instead of the dinosaurs.

    the time machine must have been built by someone else in order for it to have been stolen and then donated to the museum. — Luke
    Does not follow. That sort of reasoning is only valid if time travel is not possible. The whole point is that it was never built. — noAxioms
    You will need to explain why my objection does not follow.
    Try to state the logic of your statement formally. What are the premises? How does your conclusion (that the closed-loop machine must have been built) follow? One of your premises is perhaps that all things need creating at some point, but that premise begs a universe with no closed time curves.

    If you don't have a time machine then you can't time travel
    I accept that premise, at least for purposes of this issue.
    so you can't then obtain that time machine (or its technology) via time travel.
    Why not? It works, does it not? This is worded as a conclusion, not an additional premise. I don't accept it since 1) it doesn't follow from the premise, and 2) it is easily falsified by counterexample.

    Again, try to word it more formally, and the errors will stand out better. I think the example of learning the technology from the future works better than actually being handed the machine.

    Which hole are you referring to? Entropy?
    The one we discussed: the machine needing to exist for infinite time without showing any wear. Hence better to be handed the plans than to be handed the machine. The movie predestination works that way. It depicts a closed loop, without the infinite-age issue.

    The loop could have started only 10 years ago.
    It wouldn't be a loop if it had. Loops don't have a start.
  • Luke
    2.6k
    You're not reading my comment. I said that by your rules, a person can be in the presence of at most one actual time traveler. We could have a factory that made them like bags of cheetos, and everybody used them to get to appointments and catch the traffic light that just went yellow. If they were used like that, the planet would quickly have a population of zero in not just the original, but all the timelines. Despite that prediction, no person would ever be in the presence of more than one actual time traveler, which is the one and only person that created the specific timeline the person finds himself in (if he's still in it and hasn't left already).noAxioms

    Unless more than one person used the same time machine to time travel together. I don't understand why the planet would quickly have a population of zero in all timelines though.

    Actually, nobody would use the machines, due to the overwhelming evidence of it being nothing more than a self-annihilation machine. So good thing Bob is the only person that has one, and only Bob fails to exist in pretty much any of the timelines.noAxioms

    Bob would continue to exist on any timeline he travelled to (at least, until he dies).

    But the way you describe it, it isn't really the past, just a different timeline which maybe looks like 'the' past, but is actually just another line, 'a' past at best, one of many. There is only one 'the' past, and you didn't go there.noAxioms

    Right, but it would be logically impossible to travel to 'the past' (i.e. on a single timeline) unless I was somehow already there before I time travelled. But how could I already be there before I time travel?

    But if he's there at all, history is gone. If I go back 250 million years to see the early evolution of mammals, I'm sorry, but humans will never evolve from that timeline. Your very presence destroys that...noAxioms

    How?

    Loops don't have a start.noAxioms

    Maybe it all boils down to this. I'm arguing that causal loops require a start; that there must be an initial time travel event which causes the loop in the first place, and that what causes the initial time travel event and subsequent causal loop cannot be an effect of that initial time travel event. I don't believe that a time machine can just magically exist uncaused.

    Try to state the logic of your statement formally. What are the premises? How does your conclusion (that the closed-loop machine must have been built) follow? One of your premises is perhaps that all things need creating at some point, but that premise begs a universe with no closed time curves.noAxioms

    I've been trying to state my premises. I will try again to be more clear.

    My premises would be that:
    - one cannot time travel without a time machine
    - time machines need creating at some point
    - there must be an initial time travel event following the creation of the time machine, when the time machine is first used to time travel
    - the initial time travel event cannot cause itself (e.g. by a prior time travel event using the newly-invented time machine).
  • Bella fekete
    135



    “ As always, these are vague, young, naive ideas I’m having. I’m not trained or anything yet”




    I think this is as close as it gets, and I’ve been struck by somewhat of a structural analogy that may further the idea.


    Wittgenstein in his missing years watched movies constantly, anything on the screen. He was said to be autistic. At least on the continuum.

    In the movies, the rate of projecting stills into movement, I believe has a correlation to receiving signal which represent the frames, or bound content at which the signal projected on the screen can simulate the reality that expresses normal, everyday movement.Too fast a rate shows that as less real a representation, as too slow as well.

    As the rate of projection nears the unrecognizable blur of the continuum, the frames , begin to loose their distinct stillness of separable framings of reference,

    What happens then is that spatial perimeters elongate and trick the rate of eyeball incoming signals and the signaled neurons interpret this as a spatial increase, while duration as represented appears to increase

    .Wittgensteins understanding was difficult to interpret because his metaphysical perception , of qualifying a Kantian-categorical sense of boundedness, could
    visually only recognize it’s quantitative sequencing ,as the unreality of rates of projection could relate only to absolutely framed referenced signals without motion to the non objective look of the stop content .There must have been that, which excluded the range considered as near real representation.

    This too, is a naively real formulation of what goes on within the semblance of minds occupied with ideas such as looking back toward a journey to the back.
  • noAxioms
    1.4k
    Unless more than one person used the same time machine to time travel together.Luke
    Right. Neglected that bit.

    I don't understand why the planet would quickly have a population of zero in all timelines though.
    well, if everybody had one and knew it worked, I suppose they'd all use it and exit any particular timeline. It's sort of like heaven: The sales pitch is great, but if it's such a better place, why does nobody voluntarily hit the button and go there? It's because from the perspective of the original timeline, it just looks like you vanish, never to be seen again. There is zero evidence that it is safe, let alone works.

    Bob would continue to exist on any timeline he travelled to (at least, until he dies).
    And Bob is missing from every timeline except one. Of course on the other timelines, there may be many people that attest to having traveled, and the evidence is there that it works. Those timelines would empty out faster than the original, if only from people going back to times when there were still people to meet.
    Nobody on these worlds knows who the actual time traveler is (the one that created this world), not even Bob.

    But how could I already be there before I time travel?
    With a time machine of course. That sort of logic only holds water because there are no time machines possible.

    If I go back 250 million years to see the early evolution of mammals, I'm sorry, but humans will never evolve from that timeline.

    How?
    Evolution is a chaotic function. The popular term for it is 'butterfly effect'. The killer asteroid is not chaotic, so you don't alter that, but evolution is a random process, and you've totally altered that. People are not an inevitable result of the state of 250 million years ago. It's an inexpressibly low chance even without the traveler mucking things up. OK, that last statement presumes a lack of hard determinism. Our discussion has a lot of quantum interpretation implications as well as implications for interpretation of time. The SEP article didn't mention the former.

    Maybe it all boils down to this. I'm arguing that causal loops require a start; that there must be an initial time travel event which causes the loop in the first place
    No. The whole point of them is that they are uncaused. They'd not be a loop if they were caused. That it doesn't fit in with your notion of singular causality is irrelevant since all those rules must be discarded with reverse causality.

    My premises would be that:
    P1 - one cannot time travel without a time machine
    P2 - time machines need creating at some point
    P3 - there must be an initial time travel event following the creation of the time machine, when the time machine is first used to time travel
    P4 - the initial time travel event cannot cause itself (e.g. by a prior time travel event using the newly- invented time machine).
    I added bold labels. Let me know if I did it wrong.
    P1: I said I would accept this for this purpose, but there is no such requirement. If time travel was possible, somebody might be able to do it just by willing it. If a machine can do it, why can't a creature evolve a way to do it. The premise is something like saying you cannot get to grandma's house without a car. Well, that's false since evolution has given us a means of machineless locomotion.
    P2 is unacceptable. It's like trying to prove God by asserting that the universe needs creating at some point (which is itself a self-refuting argument). An un-created time machine does not violate any rules in a universe where time travel is possible.

    The argument falls apart there: an unacceptable premise, which again, has a counterexample which falsifies it.

    P3 seems false. I might make a time machine but never use it. We presume you mean the machine in the loop, so yes, it just happens to get used (the 'first time' say) in the story you are creating. I put 'first time' in scare quotes because there can't be a first time in a loop: there cannot be an odometer on the machine that records how many jumps it has taken.

    P4 is OK, but seemingly irrelevant since your story involves only a single time travel event, no loop at all.
    I listed them all as postulates (and no conclusions) since none are worded as conclusions. I don't see any conclusion of the impossibility of a closed loop. The whole things doesn't seem to discuss loops at all. It discusses only a created time machine, not a looping one.
  • Luke
    2.6k
    I don't understand why the planet would quickly have a population of zero in all timelines though.
    — Luke

    well, if everybody had one and knew it worked, I suppose they'd all use it and exit any particular timeline. It's sort of like heaven: The sales pitch is great, but if it's such a better place, why does nobody voluntarily hit the button and go there? It's because from the perspective of the original timeline, it just looks like you vanish, never to be seen again. There is zero evidence that it is safe, let alone works.
    noAxioms

    In that case everyone would remain on the same timeline, so it does not follow that every timeline would quickly have a population of zero.


    Bob would continue to exist on any timeline he travelled to (at least, until he dies).
    — Luke

    And Bob is missing from every timeline except one. Of course on the other timelines, there may be many people that attest to having traveled, and the evidence is there that it works. Those timelines would empty out faster than the original, if only from people going back to times when there were still people to meet.

    Nobody on these worlds knows who the actual time traveler is (the one that created this world), not even Bob.
    noAxioms

    This still doesn't explain how every timeline would quickly have a population of zero.

    But how could I already be there before I time travel?

    With a time machine of course.
    noAxioms

    What I meant was: how could I already be in the past before I have ever time travelled? I could already be in the past (on a single timeline) if I had time travelled before but, given causality, there must have been a first time that I ever used the time machine to time travel. How could I already be in the past prior to that?

    That sort of logic only holds water because there are no time machines possible.noAxioms

    Why are no time machines possible? That's not something I've said.

    They'd not be a loop if they were caused. That it doesn't fit in with your notion of singular causality is irrelevant since all those rules must be discarded with reverse causality.noAxioms

    We can just discard causality and assume that time machines don't need to have had a first ever use, and we can conveniently disregard whatever history led up to that first ever use?

    P1: I said I would accept this for this purpose, but there is no such requirement. If time travel was possible, somebody might be able to do it just by willing it. If a machine can do it, why can't a creature evolve a way to do it. The premise is something like saying you cannot get to grandma's house without a car. Well, that's false since evolution has given us a means of machineless locomotion.noAxioms

    That's fine, but if a creature evolved a way to do it, then there must have been a first time that they ever time travelled. Evolution actually works in favour of my argument because it cannot disregard the history that precedes the first ever time travel event.

    P2 is unacceptable. It's like trying to prove God by asserting that the universe needs creating at some point (which is itself a self-refuting argument). An un-created time machine does not violate any rules in a universe where time travel is possible.noAxioms

    If you accept that one cannot travel without a time machine (P1) - at least, for the sake of argument - then it follows that a time machine (or the means for time travel) must be created or have evolved or somehow brought into existence in some manner. This is not trying to prove God in order to prove the existence of the universe; it's merely assuming the universe must have been brought into existence (which is quite self-evident). Hopefuly we can agree to the standard scientific view that the universe's existence began with the big bang, but even if we might assume that the universe has always existed, this is not very much like a time machine. To say that time machines have always existed is more like saying that waffle irons have always existed.

    An un-created time machine does not violate any rules in a universe where time travel is possible.noAxioms

    Sure, but it would imply no time travel.

    P3 seems false. I might make a time machine but never use it. We presume you mean the machine in the loop, so yes, it just happens to get used (the 'first time' say) in the story you are creating. I put 'first time' in scare quotes because there can't be a first time in a loop:noAxioms

    Why can't there be a first time in a loop? Loops are immune to causality?

    P4 is OK, but seemingly irrelevant since your story involves only a single time travel event, no loop at all.noAxioms

    It's not irrelevant. I'm saying that if a loop involves time travel (as the examples in the SEP article do), then we can consider the first ever time travel event in that loop and what preceded it. Unless you are arguing that there is no causality in a loop or that time travel loops and time machines in loops have always existed? Why should causal loops be immune from causality; from having been caused? It seems like a bit of magic.
  • noAxioms
    1.4k
    so it does not follow that every timeline would quickly have a population of zero.Luke
    Agree that if the people (especially those on the original timeline) fear the readily available devices, they wouldn't get used and the population remains.

    The people on the alternate timelines would have solid evidence of it working. A car gets caught by a train crossing, so it goes back 1 minute in time and sneaks across the tracks before the gates come down. The traveler can see her own car doing it as she approaches, and the bystanders can witness it as well. The person doesn't always reappear. Sometimes the traveler is gone forever. Kind of pot luck, a game of Russian roulette.

    What I meant was: how could I already be in the past before I have ever time travelled?
    If you time travel to the past, by definition you end up somewhere 'before' the event where you initiated the travel. I kind of lost track of the context. Are we talking about the loop here?

    I could already be in the past (on a single timeline) if I had time travelled before but, given causality, there must have been a first time that I ever used the time machine to time travel.
    I don't see how that follows with the loop scenario. There would be no 'first time' to a loop. As I said, there can be no odometer on the machine counting jumps. That would be a contradiction.
    If it's just teaching the younger-self how to do it, then every jump is the only jump, so I guess that would count as the first (and only) time, or at least the one jump that defines the simple loop.

    How could I already be in the past prior to that?
    I don't understand this. If the jump is from 2024 to 1990, then 1990 is 'the past' destination, and you are not in a past that is prior to that except perhaps as a young person, the one that you teach.

    That sort of logic only holds water because there are no time machines possible.
    — noAxioms

    Why are no time machines possible? That's not something I've said.
    We're presuming they're possible, hence the logic you give being fallacious. Things that are impossible in this universe are not impossible in this alternate universe where time travel makes for different causal rules. A loop is valid under the new rules. It doesn't violate anything except the rules of this universe.

    We can just discard causality and assume that time machines don't need to have had a first ever use, and we can conveniently disregard whatever history led up to that first ever use?
    You need to discard the causality rules of this universe, yes. The rules are different in the universe we're discussing. With the loop scenario, there is no 'first ever' to it. You can't count them. The loop is just there, and is self-consistent.

    but if a creature evolved a way to do it, then there must have been a first time that they ever time travelled.
    Not if it is part of a loop. The whole 'must be a first time for everything' is only a rule in a universe like ours, intuitive to us, but not true in the sort of scenario we're discussing. Yet again, a simple counterexample falsifies your assertion. So maybe this time traveling creature never evolved, but just is. Again, there are movies depicting pretty much this.

    P2 is unacceptable. It's like trying to prove God by asserting that the universe needs creating at some point (which is itself a self-refuting argument). An un-created time machine does not violate any rules in a universe where time travel is possible.
    — noAxioms

    If you accept that one cannot travel without a time machine (P1) - at least, for the sake of argument - then it follows that a time machine (or the means for time travel) must be created or have evolved or somehow brought into existence in some manner.
    I will not. We're discussing the possibility of closed loops, and loops falsify P2.

    it's merely assuming the universe must have been brought into existence (which is quite self-evident).
    I don't find that evident at all. It violates Einstein's theories for starters, which suggests that time is part of the universe, and not something in which the universe is contained and in need of being created.

    Hopefuly we can agree to the standard scientific view that the universe's existence began with the big bang,
    Pop science view maybe. OK, if one confines one's definition of 'the universe' to just what evolved from the big bang, then a good deal of them would suggest a larger structure from which that bang was initiated. But there is no before/after without the sort of time that boiled out of the bang, so calling it 'before' is misleading.
    If you consider the universe to be the entire quantum structure, which includes all the stuff that 'springs' from it, our bang being one of them, then that structure is not something 'created' or 'caused'. It cannot be, both terms implying a larger container for something we're defining to be the largest container.

    but even if we might assume that the universe has always existed
    Sorry to be so buggy, but I don't buy that either. The phrase once again implies a universe contained by time, and not the other way around. Yes, there are those that suggest something like that, in denial of Einstein's postulates.

    To say that time machines have always existed is more like saying that waffle irons have always existed.
    Your wording suggests that the machine exists at all times, which isn't the case. It exists in the loop in the museum case. It doesn't exist at other times.
    In the teaching case, it is built by young Bob in say 2022 with knowledge from his older self. Bob can then use to travel all over the place, here and there, to say the restaurant at the end of the universe. Eventually he goes to 1990 and teaches his younger self the secrets. Then he's off again to see even more wonders. Point is, there no point in time where that machine cannot be unless it has a limited range or something.

    An un-created time machine does not violate any rules in a universe where time travel is possible.
    — noAxioms

    Sure, but it would imply no time travel.
    Same counterexample falsifies this.

    Imagine you're holding one of those party poppers that you pull and it explodes a bit of confetti around. You're about to do it and a box appears in front of you from which a some guy jumps out and explodes his own party popper as he says "three!". Then he grabs yours (unpopped), and apologizes, says the box is a time machine that goes back 8 seconds, the says "One, two, ..." and the box disappears, leaving you simply befuddled. That's what an 8 second loop looks like.
    It's not too hard to take that one apart, but not by the logic you've been attempting.

    Why can't there be a first time in a loop? Loops are immune to causality?
    The kind of causality rules you're thinking of don't exist in a universe with time travel. A first time for a loop would contradict its existence, which is travel from the other end of the loop and not somewhere else.

    P4 [the initial time travel event cannot cause itself (e.g. by a prior time travel event using the newly- invented time machine).] is not irrelevant. I'm saying that if a loop involves time travel (as the examples in the SEP article do), then we can consider the first ever time travel event in that loop and what preceded it. Unless you are arguing that there is no causality in a loop or that time travel loops and time machines in loops have always existed? Why should causal loops be immune from causality; from having been caused? It seems like a bit of magic.
    Not following. There a possibility of a loop that doesn't involve time travel? Example please.
    You're then referencing the 'first ever' go-around for a loop that cannot have such a thing. So that's what you mean by 'initial travel event'. There is no such thing for a loop, so I must withdrawm my 'is OK' assessment of it.
  • Luke
    2.6k
    What I meant was: how could I already be in the past before I have ever time travelled?
    — Luke

    If you time travel to the past, by definition you end up somewhere 'before' the event where you initiated the travel.
    noAxioms

    I was talking about the time before the first time travel event; before you've ever time travelled. You're talking about what happens if (or after) you time travel, so you're not talking about the time before you've ever time travelled.

    There would be no 'first time' to a loop. As I said, there can be no odometer on the machine counting jumps. That would be a contradiction.noAxioms

    Why can there be no odometer on the time machine counting jumps?

    That would be a contradiction.noAxioms

    What's the contradiction?

    I could already be in the past (on a single timeline) if I had time travelled before but, given causality, there must have been a first time that I ever used the time machine to time travel. How could I already be in the past prior to that?
    — Luke

    I don't understand this. If the jump is from 2024 to 1990, then 1990 is 'the past' destination, and you are not in a past that is prior to that except perhaps as a young person, the one that you teach.
    noAxioms

    Sorry to be unclear again. What I meant was: how could I be in the past as a time traveller prior to the first use of the time machine.

    Things that are impossible in this universe are not impossible in this alternate universe where time travel makes for different causal rules.noAxioms

    What are these different causal rules? There are still causes and effects, it seems. The older self can teach the younger self about time travel technology and the younger self can then use that knowledge in order to time travel from the future to the past. Or, the younger self can steal a time machine from the museum and then later use that time machine in order to donate the time machine back to the museum. The only different causal rule appears to be that there can be no first time travel event or that we are not allowed to talk about the first time travel event, for some unspecified reason.

    So some causal rules are okay, but not others? We may never ask/explain how a time machine came into existence in the universe, but it's okay to ask/explain how a time machine came into existence in a museum?

    The rules are different in the universe we're discussing. With the loop scenario, there is no 'first ever' to it. You can't count them. The loop is just there, and is self-consistent.noAxioms

    It's logically self-consistent as long as we never consider a loop as having a first time travel event or what preceded it, it seems. But how is it logically self-consistent that there was never a first time travel event? Does time or causality work differently in these scenarios such that it would be impossible to trace back to the first time travel event? If so, then how do time or causality work differently? Why is it impossible to trace back to the first time travel event? Or, why can't there be a first time travel event?

    Not if it is part of a loop. The whole 'must be a first time for everything' is only a rule in a universe like ours, intuitive to us, but not true in the sort of scenario we're discussing.noAxioms

    How is it "not true"? It doesn't seem to me that it's not true; it seems that you just want me to ignore it.

    So maybe this time traveling creature never evolved, but just is. Again, there are movies depicting pretty much this.noAxioms

    We're dispensing with evolution, too? I'm unfamiliar with those movies/scenarios.

    The scenarios in the SEP article and those I've been considering all involve human time travel with a time machine. I suppose I could alter P2 to say that time machines involved in human time travel need to be created at some point. Or, better still, P2 could say that there must be a first human time travel event associated with the human use of a time machine or time travel device/technology (assuming that any such events occur).

    I will not. We're discussing the possibility of closed loops, and loops falsify P2.noAxioms

    You are effectively telling me to ignore how the time machine came into existence originally. That doesn't "falsify" P2.

    To say that time machines have always existed is more like saying that waffle irons have always existed.
    — Luke

    Your wording suggests that the machine exists at all times, which isn't the case. It exists in the loop in the museum case. It doesn't exist at other times.
    noAxioms

    There is no problem in saying that time machines or waffle irons have always existed in a causal loop, which is what you suggest when you say they are uncaused. This wording need not imply that these things exist at all times. My point, however, was that time machines, like waffle irons, are man-made for a particular purpose, and so it seems unlikely that they would naturally exist without any human (or other sentient beings') intervention. Also, I note that we were talking about human time travel using time machines until recently, but now you're invoking fanciful beings that can time travel without any time machines and other magical shenanigans in order to try and save the "self-consistent" logic of causal loops.

    An un-created time machine does not violate any rules in a universe where time travel is possible.
    — noAxioms

    Sure, but it would imply no time travel.
    — Luke

    Same counterexample falsifies this.

    Imagine you're holding one of those party poppers that you pull and it explodes a bit of confetti around. You're about to do it and a box appears in front of you from which a some guy jumps out and explodes his own party popper as he says "three!". Then he grabs yours (unpopped), and apologizes, says the box is a time machine that goes back 8 seconds, the says "One, two, ..." and the box disappears, leaving you simply befuddled. That's what an 8 second loop looks like.
    noAxioms

    Am I supposed to be the guy in the box/time machine, because this doesn't sound like a causal loop; it's just a guy using a time machine to go back in time every 8 seconds to do the same thing repeatedly. It's unlike the other causal loop scenarios because it's not clear that I ever become the guy in the box/time machine. Or was that part left unsaid?

    Anyhow, I thought by "un-created" you meant that the time machine was not created or did not exist. Did you mean "uncaused"?

    The kind of causality rules you're thinking of don't exist in a universe with time travel. A first time for a loop would contradict its existence, which is travel from the other end of the loop and not somewhere else.noAxioms

    Right, that's why I've been arguing that time travel only makes sense on multiple timelines, and why I've been arguing that a causal loop (or that time travel on a single timeline) does contradict its existence. I don't see why I should ignore there being a first time travel event just for the sake of maintaining the consistency of a causal loop. I'm arguing that they do contradict their existence.

    Not following. There a possibility of a loop that doesn't involve time travel? Example please.noAxioms

    No, I was just trying to restrict it only to causal loops that do involve time travel, in case you were about to bring up any causal loops that don't.
  • noAxioms
    1.4k
    I was talking about the time before the first time travel event; before you've ever time travelled. You're talking about what happens if (or after) you time travel, so you're not talking about the time before you've ever time travelled.Luke
    OK, Bob makes the machine and uses it to go from 2024 to a new timeline starting at 1990. Any point on the original timeline before Bob vanishes from it is the time before the first travel event. There is no time on the new timeline before the first travel since it starts there, kind of per last-Tuesdayism.
    I lost track of the question about this 'time before'. Are we talking about say 2023 on the original timeline or am I still getting it wrong?

    Why can there be no odometer on the time machine counting jumps?
    There can't be one on the machine that jumps in the loop. Bob's machine can have an odometer, no problem.
    The contradiction: Suppose, just before the jump, the odometer reads x. It arrives at its destination (8 seconds in the past in my popper example) and immediately increments the thing to x+1. This contradicts it leaving 8 seconds later with a reading of x.

    Sorry to be unclear again. What I meant was: how could I be in the past as a time traveller prior to the first use of the time machine.
    Just repeating the same question doesn't make it clear. Are we talking about Bob and the copy-timeline scenario? If so, you need to specify which timeline you're referencing when talking about one thing being prior to another.
    In general, if one considers that Bob builds the machine and first uses it in 2024 and uses it to go to 'the past' (no timeline specified), then since 'the past' is typically considered to be prior to 'the present', Bob is in the past as a time traveler (in 1990 or whatever) prior to 2024 since 1990 is often considered to be prior to 2024. So that's how he's in the past prior to first using the thing. It's the whole point of the machine to be able to do this.

    What are these different causal rules?
    That's what you are apparently trying to figure out. I don't know either, so I'm also exploring. What I don't do is presume the usual rules, such as that a place that almost looks like the state of things in 1990 is prior to the state of things in 2024. I also don't presume that the cause of a thing is necessarily prior to the thing. That's a pretty obvious one to throw out.

    There are still causes and effects, it seems.
    Agree. We're trying to keep that. The loop is causally closed, so I don't see it as a contradiction. The cause of the 8-second guy is his own travel event 8 seconds later.

    The older self can teach the younger self about time travel technology and the younger self can then use that knowledge in order to time travel from the future to the past. Or, the younger self can steal a time machine from the museum and then later use that time machine in order to donate the time machine back to the museum. The only different causal rule appears to be that there can be no first time travel event or that we are not allowed to talk about the first time travel event, for some unspecified reason.
    There is no first time for the loop, or if there is, it's the only time. There is after all but the one jump, per the external timeline, presuming its a simple loop. Only the machine's timeline has multiple jumps, plus its contents if those contents go from arrival all the way back into the machine at departure.

    So some causal rules are okay, but not others?
    The ones not OK lead to contradictions. The looping machine having its own 'first time' leads to a contradiction. It would effectively be an odometer going from 0 to 1, and we showed how that is a contradiction.

    We may never ask/explain how a time machine came into existence in the universe
    It came into existence by traveling from 'the future'. You can ask and that's the answer. That universe allows that sort of causality.

    but it's okay to ask/explain how a time machine came into existence in a museum?
    By being donated of course.

    I think the rule you find hard to discard is that all seemingly artificial things must somehow be invented and assembled at some point, and the examples we show are consistent without all those steps. Sure, the machine is built in the teaching loop, but the technology knowledge (the inventing) is the loop, information that is never gleaned, but is merely passed on.

    It's logically self-consistent as long as we never consider a loop as having a first time travel event or what preceded it, it seems.
    You're trying to find a logical inconsistency, and I don't see one. Before the loop, the machine simply doesn't exist, nor does it after. The 8-second machine exists but for 8 seconds. Not time to study and figure out how its done, something the museum guys might decide to attempt.

    But how is it logically self-consistent that there was never a first time travel event?
    The same way that the lack of the most eastern point isn't a logical inconsistency? It's only inconsistent if you presume there must be a first time (on the machine's timeline), so that's apparently a wrong thing to presume. There's a first time on the world's timeline. Isn't that enough? This presumes that the external world is itself not a loop. There are hypotheses that suggest otherwise, a sort of cyclic model of the universe.

    Does time or causality work differently in these scenarios such that it would be impossible to trace back to the first time travel event?
    The infinite-age universe hypothesis similarly suggests the impossibility of tracing back to a first event. A loop without a beginning is not in contradiction with anything.

    Not if it is part of a loop. The whole 'must be a first time for everything' is only a rule in a universe like ours, intuitive to us, but not true in the sort of scenario we're discussing.
    — noAxioms

    How is it "not true"? It doesn't seem to me that it's not true; it seems that you just want me to ignore it.
    It's true in our universe because I cannot think of a scenario where at some earlier time there is not a mug, and at a later time there is a mug, and that there it a beginning to the mug's timeline. The timeline of the mug and that of the rest of the universe is completely parallel, so there must be a 'first moment' for it. In this alternate universe, the mug timeline might not be parallel. It still has a first (and only) time in the universe timeline, but not on its own timeline, which isn't parallel to the one 'outside'.

    We're dispensing with evolution, too?
    If a machine that loops and is never created can exist in some consistent way, then so can a creature than has no evolutionary ancestory. It just appears from some retrocausal event, and its existence somehow eventually plays a role in that eventual retrocausal event.
    So it's like humans have no evolutionary ancestors (despite the biology folks suggesting otherwise). But some time far in the future, when humanity is near its end but they develop time travel, the put a couple back to year -4000 and name them Adam & Eve.
    Geez, the religious folks would jump on this if it wasn't supposed to be blamed on God instead of retrocausality.

    I suppose I could alter P2 to say that time machines involved in human time travel need to be created at some point.
    OK. The 8 second machine is created in front of me at some point, and un-created 8 seconds later when it vanishes. Works for me.

    Or, better still, P2 could say that there must be a first human time travel event associated with the human use of a time machine or time travel device/technology (assuming that any such events occur).
    Dangerous to use the word 'first' when the temporal ordering of things is not objective. I think that's where a lot of the trouble comes from.

    You are effectively telling me to ignore how the time machine came into existence originally.
    Nope. It came into existence when it first appears, not 'uncaused'. It doesn't exist at any time before that, so that is it coming into existence. It gets donated to the museum some time later and yet later is stolen and vanishes from existence forever after as it causes the earlier event.

    now you're invoking fanciful beings that can time travel without any time machines and other magical shenanigans in order to try and save the "self-consistent" logic of causal loops.
    I just made them up as another example which isn't directly self contradictory.

    I should probably withdraw my language that these things are not created. They are, but the causes of their creation are events that are future events as measured by the world-outside timeline, a timeline which we are presuming to be reasonably linear and therefore orderable.

    Am I supposed to be the guy in the box/time machine, because this doesn't sound like a causal loop
    No, you are the spectator who has somebody use your popper and then take it from you. The person in the box is, well ... something else. It is along those lines that you should tear this apart. A human makes a great odometer, and you can't have an odometer, so the guy is perhaps not human?

    it's just a guy using a time machine to go back in time every 8 seconds to do the same thing repeatedly. It's unlike the other causal loop scenarios because it's not clear that I ever become the guy in the box/time machine. Or was that part left unsaid?
    Yes to the first. No, it's never you. You're left behind being befuddled, remember? You never see him again. It very much is a loop, and a very tight one.

    I thought by "un-created" you meant that the time machine was not created or did not exist. Did you mean "uncaused"?
    Un-create means to cease existing. From the perspective of the linear timeline, Any traveler uncreates his machine and himself. It's just gone leaving not even disassembled parts. Of course on the machine's timeline, it just has an external environment change and isn't an act of creation or uncreation at all.

    Somebody could catch a video of uncreation on their phone. You're were taking a video of a train crossing because you heard a train in the distance. The video records a blue car suddenly appearing (an act of spontaneous creation) at the crossing which then crosses and continues on. The lights go on and the gates come down, just as an identical blue car pulls up and vanishes (uncreates) at the exact spot where the prior car appeared. You have a video of time travel in action, and it even worked in the universe of Bob where a new timeline is created each time. The video is subsequently sold to the time machine sales people who use it to pitch their product.

    Right, that's why I've been arguing that time travel only makes sense on multiple timelines
    For the most part I agree. But single-timeline travel isn't necessarily contradictory so long as one does not make choices known to be different than those made before. It does require a sort of lack of free will as it is often defined.

    I was just trying to restrict it only to causal loops that do involve time travel, in case you were about to bring up any causal loops that don't.
    Any loop in time is contrary to the sort of linear ordering of all events that we find intuitive. No, it doesn't have to be labeled 'time travel'. A cyclic universe is a nice loop that isn't considered time travel because there is no linear timeline laid alongside the loop.
  • ssu
    8.2k
    Time travel to the past hypothetically possible?

    Hypothetically yes.

    Theoretically? That's the question.

    Realistically? Let's gain the ability to travel 99,9% of the light speed first...
  • Luke
    2.6k
    OK, Bob makes the machine and uses it to go from 2024 to a new timeline starting at 1990. Any point on the original timeline before Bob vanishes from it is the time before the first travel event. There is no time on the new timeline before the first travel since it starts there, kind of per last-Tuesdayism.
    I lost track of the question about this 'time before'. Are we talking about say 2023 on the original timeline or am I still getting it wrong?
    noAxioms

    Yes, we are talking about the original timeline. As I said earlier:

    ...it would be logically impossible to travel to 'the past' (i.e. on a single timeline) unless I was somehow already there before I time travelled. But how could I already be there before I time travel?Luke

    I think you've acknowledged this is not possible. It is why I was arguing that more than one timeline is required.

    There can't be one on the machine that jumps in the loop. Bob's machine can have an odometer, no problem.
    The contradiction: Suppose, just before the jump, the odometer reads x. It arrives at its destination (8 seconds in the past in my popper example) and immediately increments the thing to x+1. This contradicts it leaving 8 seconds later with a reading of x.
    noAxioms

    Sorry, but I don't see the contradiction. The time machine starts with an odometer reading of x (representing the number of time travel "trips" made by the time machine), then it makes a (another) time travel "trip" and records a reading of x+1. Why does it need to revert to a reading of x again 8 seconds later? Isn't it correct that the number of "trips" made by the time machine has increased from x to x+1?

    That's what you are apparently trying to figure out. I don't know either, so I'm also exploring. What I don't do is presume the usual rules, such as that a place that almost looks like the state of things in 1990 is prior to the state of things in 2024. I also don't presume that the cause of a thing is necessarily prior to the thing. That's a pretty obvious one to throw out.noAxioms

    On reflection, I want to reject my suggestion that there is more than one timeline. You've helped me to see that this is not really what I had in mind. What I have in mind is that there is only a single timeline but that the effects of the first time travel event overwrite the past of the original timeline (starting from the destination time of the time travel event, e.g. 1990). This might create a causal loop or it might not. However, the main idea I've been trying to convey all along is that there must be an original version of "the past" prior to the first ever time travel event, which gets overwritten and is necessarily different to the version of "the past" that exists post-time travel. This helps to retain "normal" causality, thus removing the need for the magical appearance of time machines or technology "out of nowhere", existing uncaused (as in a causal loop) and thus removes the impossibility of killing one's own grandfather (as in the grandfather paradox). It also removes the unpopular idea that time travel creates a "copy" of the original timeline. And it retains free will.

    In fact, causal loops can be avoided in the examples we've discussed because the time traveller can just do something differently at any time after any of their time travel events. For example, there is nothing forcing the 8-second guy to time travel again after 8 seconds.

    I think the rule you find hard to discard is that all seemingly artificial things must somehow be invented and assembled at some point, and the examples we show are consistent without all those steps. Sure, the machine is built in the teaching loop, but the technology knowledge (the inventing) is the loop, information that is never gleaned, but is merely passed on.noAxioms

    Perhaps they're logically consistent, but I find the magical, uncaused existence of the causal loop universes (and their contents) to be too far-fetched and unsatisfactory. Someone who gets their time travel technology knowledge from their older time travelling self does not explain how the older time travelling self got this knowledge in the first place, before they first time travelled. And I find the idea that there is no first time travel event unrealistic.

    I suppose I could alter P2 to say that time machines involved in human time travel need to be created at some point.
    — Luke

    OK. The 8 second machine is created in front of me at some point, and un-created 8 seconds later when it vanishes. Works for me.
    noAxioms

    That doesn't explain how the time machine was created for the time traveller.

    Dangerous to use the word 'first' when the temporal ordering of things is not objective. I think that's where a lot of the trouble comes from.noAxioms

    I'm not using 'first' as an objective ordering of things but as a human ordering of things. I don't assume the determination of a 'first' event is independent of some group of language users (particularly, English speakers).

    Nope. It came into existence when it first appears...noAxioms

    It has a first appearance?

    But single-timeline travel isn't necessarily contradictory so long as one does not make choices known to be different than those made before.noAxioms

    But is single-timeline travel necessarily contradictory, even if one does make different choices post-time travel?
  • noAxioms
    1.4k
    As I said earlier:
    ...it would be logically impossible to travel to 'the past' (i.e. on a single timeline) unless I was somehow already there before I time travelled. But how could I already be there before I time travel?
    — Luke

    Yes, we are talking about the original timeline.
    Luke
    Given physics where there is a timeline that is the original one, that line cannot have a time traveler in it at all. All the copy lines have but the traveler(s) that created that line (assuming the machine had one or more passengers). So in those lines, any traveler was already there at its start.

    Given the physics of a single timeline, various machines might travel here and there, but there would be presumably some earliest one (to see dinosaurs say). In such a situation, there is no traveler before that earliest Cretaceous period. I don't think you're asking that, but who knows...

    There are valid scenarios with such a single timeline, but the traveler (if it is human) is part of 'the past' then and lacks the free will to do otherwise. I agree this runs into serious problems if he knows what he does (say a minute ago) and goes back explicitly to do a different thing. No amount of determinism is going to prevent that. Determinism is not a thing with a will different than yours. Nobody seems to realize that.

    Why does it need to revert to a reading of x again 8 seconds later?
    Because it reads x when it appears 8 seconds before that. You know that. It's on the outside and you read it. You can't read it being x, x+1, and all the other numbers. The number has to match at both ends, or it didn't come from that 8-seconds hence jump. It wouldn't be a loop, just a stopover, and a different party popper than the one he took from you.

    On reflection, I want to reject my suggestion that there is more than one timeline. You've helped me to see that this is not really what I had in mind. What I have in mind is that there is only a single timeline but that the effects of the first time travel event overwrite the past of the original timeline (starting from the destination time of the time travel event, e.g. 1990).
    OK. That's not something we discussed yet. How does it empirically differ from the branch thing? The old 'history' goes away, so there's nobody to witness the population of the world going down. There are a whole mess of uncaused events going on, but besides the classical impossibility of that, no other contradictions. You have people who don't have valid identification. Maybe no other people at all. So the empirical experience of those you don't take with you is irrelevant, and the empirical experience of the traveler is identical to the experience of the guy in the branching model. So this sounds like a different interpretation of the exact same experience.

    Those are just my thoughts before going on to read what you said about it:

    This might create a causal loop or it might not. However, the main idea I've been trying to convey all along is that there must be an original version of "the past" prior to the first ever time travel event
    OK, back before the earliest time, before the destination of any retro-time traveler.
    Or do you mean 'first' on the timeline of some traveler instead of on the one world timeline? You don't ever specify. I think you mean the latter, in which case, what do you mean by 'must be'? How can something 'be' if it doesn't exist at any time? What definition of 'be' are you using?

    and is necessarily different to the version of "the past" that exists post-time travel.
    Suppose I travel to 1990. How is what you call the original line (the one I remember with cellphones and all) is 'the past'? It's not before 1990, and for that matter, it's not after either. It just isn't at all.

    It all sounds like a re-growing-block model, except that disallows forward time travel since the destination specifed doesn't yet exist. I set sights for the year 3000 (like in Futurama), but while my machine is waiting for Y3000 to come around, somebody else uses a time machine to go back to 1985, thus obliterating me and the destination I targeted. same fate awaiting all those people paying for cryonic preservation. It requires a stability that just isn't there.

    This helps to retain "normal" causality, thus removing the need for the magical appearance of time machines or technology "out of nowhere", existing uncaused
    But all your scenarios describe exactly that, including pretty much every fictional story that I can think of. Time travel, as envisioned, necessitates technology or at least some object/person appearing uncaused from nothing, or worse, replacing what was otherwise at that spot. Remember terminator? This sphere of space replaces what was there with some air and a naked person. Nobody says what happens to the stuff that unfortunately happened to be where that ball appears, which by chance might possibly be half of another naked person.

    And it retains free will.
    Well, the paradoxes are gone at least. Nobody can demonstrate the typical definition of free will.

    In fact, causal loops can be avoided
    They'd not be loops at all then. The 8-second guy would simply die in moments the same way the half-of-naked person did. It's a hazard of living in such a world is that your life expectancy outside the machine is moments at a time, and no better inside the machine since no time passes in there (unless you assert otherwise I guess).

    OK. The 8 second machine is created in front of me at some point, and un-created 8 seconds later when it vanishes. Works for me.
    — noAxioms

    That doesn't explain how the time machine was created for the time traveller.
    It's not created for the time traveler any more than the time traveler is created or has an age.

    It has a first appearance?
    8 second guy has a first and only appearance, yes. From his looping timeline, there is no first anything. It's a circular timeline.

    But is single-timeline travel necessarily contradictory, even if one does make different choices post-time travel?
    What you describe above is a single preferred timeline scenario, with all the non-preferred timelines being nonexistent. I am not sure if there are 'different choices' involved since there is but the one timeline, and thus one choice being made at any point in time. Sure, you remember making different choices, but those are memories of nonexistent times.
  • Luke
    2.6k
    As I said earlier:
    ...it would be logically impossible to travel to 'the past' (i.e. on a single timeline) unless I was somehow already there before I time travelled. But how could I already be there before I time travel?
    — Luke

    Yes, we are talking about the original timeline. — Luke

    Given physics where there is a timeline that is the original one, that line cannot have a time traveler in it at all. All the copy lines have but the traveler(s) that created that line (assuming the machine had one or more passengers). So in those lines, any traveler was already there at its start.
    noAxioms

    It looks like we're saying the same thing.

    However, this does not explain how a time traveller can have travelled to the past before their first ever time travel event. Bob will one day grow up to build a time machine, but before he has ever grown up or built his time machine, his older self appears from the future in a time machine. If this is a causal loop, then where are the causes or the determinism that brought the time machine into existence? There must have been an original version of the past before Bob ever created his time machine in which he was not visited by his older self.

    Given the physics of a single timeline, various machines might travel here and there, but there would be presumably some earliest one (to see dinosaurs say). In such a situation, there is no traveler before that earliest Cretaceous period.noAxioms

    In the case of this dinosaur visitor, surely this person had to have been born before they could ever travel to the past? Therefore, there must have been an original version of the past that existed before the dinosaur visitor ever visited. How could the time traveller be in that past time before they had ever time travelled or before they were born in the first place? Surely, their birth must precede all the other events of their life, including their time travel event. If their birth must precede all the events of their life (given determinism and causality), then there must be a (different) version of history without a dinosaur visitor which exists until this person was born and subsequently time travelled to visit the dinosaurs. This original version of history without a time traveller gets overwritten following the time travel event, but must have existed.

    There are valid scenarios with such a single timeline, but the traveler (if it is human) is part of 'the past' then and lacks the free will to do otherwise. I agree this runs into serious problems if he knows what he does (say a minute ago) and goes back explicitly to do a different thing. No amount of determinism is going to prevent that. Determinism is not a thing with a will different than yours. Nobody seems to realize that.noAxioms

    If you consider history from the end of time, at the end of history, then obviously nobody has the free will to change any of that. Nothing more can be done.

    Why does it need to revert to a reading of x again 8 seconds later?
    — Luke

    Because it reads x when it appears 8 seconds before that. You know that. It's on the outside and you read it. You can't read it being x, x+1, and all the other numbers. The number has to match at both ends, or it didn't come from that 8-seconds hence jump. It wouldn't be a loop, just a stopover, and a different party popper than the one he took from you.
    noAxioms

    If it's a causal loop, then it will repeat the same time travel event over and over again. However, the odometer reading of "x" (jumps) is after the time travel event. Therefore, if the odometer actually works, then its reading before the time travel event must have been "x-1" (jumps). This may not be part of the causal loop, but it logically (and causally) must be the case.

    OK. That's not something we discussed yet. How does it empirically differ from the branch thing? The old 'history' goes away, so there's nobody to witness the population of the world going down. There are a whole mess of uncaused events going on, but besides the classical impossibility of that, no other contradictions. You have people who don't have valid identification. Maybe no other people at all. So the empirical experience of those you don't take with you is irrelevant, and the empirical experience of the traveler is identical to the experience of the guy in the branching model. So this sounds like a different interpretation of the exact same experience.noAxioms

    Sure, but I'm considering the possibility of time travel, not trying to save lives or the stability of the timeline.

    This might create a causal loop or it might not. However, the main idea I've been trying to convey all along is that there must be an original version of "the past" prior to the first ever time travel event
    — Luke

    OK, back before the earliest time, before the destination of any retro-time traveler.
    Or do you mean 'first' on the timeline of some traveler instead of on the one world timeline?
    noAxioms

    I am referring to pre-time travel; before the time travel event has ever occurred. Before anyone has ever time travelled, history will be a particular way, and this particular way (or version of history) will be altered by the time travel event to create a different version of history.

    We might say or believe that up until now there have been no time travel events. If I were to time travel tomorrow, back to 1985, then I would be altering history as we now know it. After that, history will contain my time travel event, but it must also contain the "unaltered" history that preceded my time travel event (the history as we presently know it, before any time travel events). My time travel event causes the alteration of the original history, so this is consistent with causality and determinism.

    and is necessarily different to the version of "the past" that exists post-time travel.
    — Luke

    Suppose I travel to 1990. How is what you call the original line (the one I remember with cellphones and all) is 'the past'? It's not before 1990, and for that matter, it's not after either. It just isn't at all.
    noAxioms

    Not sure that I understand what you mean here, but in order to make sense of your "travel to" 1990, you cannot already have arrived at 1990 (as a time traveller) before you have time travelled to 1990.

    It all sounds like a re-growing-block model, except that disallows forward time travel since the destination specifed doesn't yet exist. I set sights for the year 3000 (like in Futurama), but while my machine is waiting for Y3000 to come around, somebody else uses a time machine to go back to 1985, thus obliterating me and the destination I targeted. same fate awaiting all those people paying for cryonic preservation. It requires a stability that just isn't there.noAxioms

    To simplify matters, we might only consider one time travel event rather than several. Also, in this discussion I'm interested in the possibiilty and consequences of time travel, not in preserving the stability of the population or the timeline.

    This helps to retain "normal" causality, thus removing the need for the magical appearance of time machines or technology "out of nowhere", existing uncaused
    — Luke

    But all your scenarios describe exactly that, including pretty much every fictional story that I can think of.
    noAxioms

    Obviously, a time machine will appear in the past to come "out of nowhere" following the time travel event. That is not what I am talking about. I am talking about the history of a time machine's construction being erased in a causal loop, such as in the museum donation scenario. This is the sort of appearance from "out of nowhere" that I am referring to; that a time machine or its technology comes to exist without any causal history. The same applies to the dinosaur visitor who can exist in the past (which is necessarily post-time travel) prior to ever having been born (which is necessarily pre-time travel).

    Nobody can demonstrate the typical definition of free will.noAxioms

    Can somebody demonstrate the typical definition of determinism?

    It's a hazard of living in such a world is that your life expectancy outside the machine is moments at a timenoAxioms

    Ain't that the truth.

    and no better inside the machine since no time passes in there (unless you assert otherwise I guess).noAxioms

    I imagine the time travel duration to be instantaneous, but I don't think it's important.

    OK. The 8 second machine is created in front of me at some point, and un-created 8 seconds later when it vanishes. Works for me.
    — noAxioms

    That doesn't explain how the time machine was created for the time traveller.
    — Luke

    It's not created for the time traveler any more than the time traveler is created or has an age.
    noAxioms

    But we can imagine how a causal loop can come to exist. For example, I spend my life working out time travel technology and build a working time machine. I then time travel back to 1990 and teach my younger self how to time travel. My younger self grows up, uses the knowledge to time travel back to 1990, and teaches my/their younger self how to time travel. A causal loop follows the initial time travel event, but it has a different history prior to the first time travel event (an original history in which I figured out time travel without having been taught it by my time travelling self).

    It has a first appearance?
    — Luke

    8 second guy has a first and only appearance, yes. From his looping timeline, there is no first anything. It's a circular timeline.
    noAxioms

    Is that a "yes" or a "no" on the first?

    But is single-timeline travel necessarily contradictory, even if one does make different choices post-time travel?
    — Luke

    What you describe above is a single preferred timeline scenario, with all the non-preferred timelines being nonexistent. I am not sure if there are 'different choices' involved since there is but the one timeline, and thus one choice being made at any point in time. Sure, you remember making different choices, but those are memories of nonexistent times.
    noAxioms

    But logically (and causally), those non-existent times did exist, prior to the time travel event.
bold
italic
underline
strike
code
quote
ulist
image
url
mention
reveal
youtube
tweet
Add a Comment

Welcome to The Philosophy Forum!

Get involved in philosophical discussions about knowledge, truth, language, consciousness, science, politics, religion, logic and mathematics, art, history, and lots more. No ads, no clutter, and very little agreement — just fascinating conversations.